« first day (42 days earlier)      last day (532 days later) » 
00:00 - 16:0016:00 - 00:00

4:03 PM
0
Q: Prove the relation between $col(AB)$ and $col(A)$ and $col(B)$

darkmoorCould you please help to prove that $col(AB) \subseteq col(B)$ and $col(AB) \subseteq col(A)$ given $Ax=b$ and $Bx=c$ and $ABx=d$ or in other words given the column spaces $col(A)$ and $col(B)$ and $col(AB)$?

0
Q: How do you solve the following (logs)

D.LooI just have a fast question. How do you solve the following? Intuition? 0.8log(9-0.2x)+0.2log(2+0.8x)=0.6log(10-0.2x)+0.4log(3+0.8x) Thanks

0
Q: A name for this property?

M.GLet $*$ be an operation such that $(xy)^* = y^*x^*$, e.g. if $xy$ are 2-by-2 matrices and $*$ is "take the inverse". Is there a name for such a property ?

Tag (terminology) should not be the only tag a question has. Please add a tag for a subject area to which the question belongs. (from a bot)Normal Human 21 secs ago
0
Q: $K(G) \leq K(G-e)$ $K(G)$ is domination number of $G$ $ K(G-e)$ is domination number of $G$

ghazaleprove thath $K(G) \leq K(G-e)$. $K(G)$ is domination number of $G$ $ K(G-e)$ is domination number of $G$ $e$ is an edge of $G$

0
Q: residue of $\frac{1}{1+z^{2}}$

nanopotatoI first expanded the equation into $\frac{1}{(z-i)(z+i)}$, then I apply the theorem that $$\underset{z=i}{Res}\frac{1}{(z-i)(z+i)}=\lim_{z\rightarrow i}(z-i)\frac{1}{(z-i)(z+i)}=\frac{1}{2i} $$ but the correct answer should be $\frac{-i}{2}$, which step is wrong?

0
Q: How to show that an odd function always goes through zero?

mathNewbieI have the standard definition of an odd-function from wikipedia: Again, let f(x) be a real-valued function of a real variable. Then f is odd if the following equation holds for all x and -x in the domain of $-f(x) = f(-x)$ Can anyone help me how to do this? Do I have to show, that it...

Consider replacing (analysis) with a more specific tag for the relevant branch of analysis. (autocomment)Normal Human 21 secs ago
0
Q: complex numbers such that $z^\alpha w^\alpha$ and $(zw)^\alpha)$ have different principal values?

user298651Is it possible to find two complex numbers $w,z$ and a complex exponents $\alpha$ such that the principal values of $z^\alpha w^\alpha$ and $(zw)^\alpha)$ are different?

 
4:31 PM
0
Q: How to solve the equation of sqrt(x)+sqrt(y)=sqrt (N) (where N=2205) in integers?

AlexanderHow to solve the equation of sqrt(x)+sqrt(y)=sqrt (N) (where N=2205) in integers? How in general to solve the similar equations?

Welcome to Math.SE, Alexander. Consider adding a tag for a broader subject area to which the question belongs. Some of these tags might fit. (from a bot)Normal Human 21 secs ago
0
Q: Linear Programming Problem with odd objective function

WissilI have the linear problem as it follows. I have 3 different types of devices. Type A, Type B, Type C. At any given moment, there is exactly one type of each device installed. So one device A, one device B and one device C. Each type of device has its own probability to get damaged, so I want to...

0
Q: calculate $\underset{n\rightarrow\infty}{lim}\left(\cfrac{1}{n^{2}}+\cfrac{2}{n^{2}}+...+\cfrac{n}{n^{2}}\right) = ?$

GuestI need help calculate : $\underset{n\rightarrow\infty}{lim}\left(\cfrac{1}{n^{2}}+\cfrac{2}{n^{2}}+...+\cfrac{n}{n^{2}}\right)$

0
Q: resolution of a nonlinear equation by using feedback

Ahmed AbrousI have a nonlinear equation, that I would like to resolve. So I would like to use a PI controler for that. Is it possible to make my equation as a feedback and to use a PI controler? Thank's.

Consider adding a tag for a broader subject area to which the question belongs. Some of these tags might fit. (autocomment)Normal Human 21 secs ago
0
Q: what does this qusetion mean?

Ashok Singh Bhandarilet (X,d) be a metric space. which of the following is possible? (A) X has exactly 3 dense subsets (B) X has exactly 4 dense subsets (C)X has exactly 5 dense subsets (D) X has exactly 6 dense subsets

0
Q: How do you solve the following (logs)

Johnderee1how do you solve the following? My intuition tells me to take exp and then raise it to five to get a polynomial. But, it stops there. Any ideas? 0.8ln(9-0.2x)+0.2ln(2+0.8x)=0.6ln(10-0.2x)+0.4ln(3+0.8x)

Welcome to Math.SE, Johnderee1. This site uses MathJax formatting of formulas. More tips here. (from a bot)Normal Human 21 secs ago
0
Q: Conceptual problem concerning equality a.e. and conditional expectation

KolminI have a conceptual problem with the relation between two integrable functions that are equal a.e.. Here there is a possible setting, to make things more concrete. Let $(X, \Sigma, \mu)$ a measure space, and let $\phi \in [0,1]^X$ be a measurable (and clearly integrable) function, such that for ...

0
Q: How to integrate $\int \sqrt{1-sin(x)} \ dx$

bru1987How can I integrate $$\int \sqrt{1-sin(x)} \ dx$$ Can't seem to figure it out. Thank you.

0
Q: $\int{ \sin^3 x cos^4 x}~dx $ by using substitution u=cos x

AmeetIntegrate sin^3xcos^4x dx by substitute u=cos x Can anyone help me cuz i am new to this integration. A little show on working step much appreciate

0
Q: Is $(-2)^{\sqrt{2}}$ a complex number and why?

Puzzled417 Is $(-2)^{\sqrt{2}}$ a complex number and why? Is there some reason if this is a complex number why we can't define it to be an integer?

0
Q: Continuity argument in Meinsma's proof of the Routh-Hurwitz test

Edwin93For a project about Systems Theory we need to study the proof of the Routh-Hurwitz test by Meinsma, 1995. The theorem says the following. A nonconstant polynomial $p(s)=p_0s^n+p_1s^{n-1}+\ldots+p_n$ with $p_i\in\mathbb{R}$ and $p_0$ not equal to zero is stable if and only if $p_1$ is nonzero, $p_...

 
4:59 PM
-2
Q: Remind forgotten questions

SharcouxIt is very boring when people ask a question and then forget about it and never select any answer. It is also impossible to differentiate obsolete questions that nobody cares about from old questions for which the asker still hopes for an answer. I may have a simple solution for this. 2 weeks af...

 
0
Q: Project selection problem (max-flow/min-cut)

fallingEntropyI try to understand the Max-Flow-Min-Cut Theorem, but somewhat fail to apply it to the Project Selection Problem. My difficulty can be illustrated by the example in this wikipedia article: https://en.wikipedia.org/wiki/Max-flow_min-cut_theorem#Project_selection_problem Graph Three projects $(...

0
Q: Modular arithmetic and Fermats Last Theorem.

LuzziHej out there. I have a assignment, and two of the questions are bugging me. 1. show that in the equation x^n + y^n =z^n have a (none trivial) whole number solution, so x^n + y^n =z^n ( mod p) for a prim number p, but not the other way around 2. show that Fermats last theorem, can be shown by x...

Welcome to Math.SE, Luzzi. This site uses MathJax formatting of formulas. More tips here. (autocomment)Normal Human 21 secs ago
0
Q: Peano Arithmetic: is this formalized statement correct?

litmusUsing Peano Axioms I have formalized the following: x is the square of an odd prime number For some odd prime number x' , x is its square IF x is some odd prime number, THEN x is the square of x' IF x' is some odd number AND x' is some prime, THEN x is the square of x' [x' is some odd number AN...

 
0
Q: "There is no language C/C++"

RhymoidSomething that regularly demotivates me from participating on the c tag is that when someone tags their question with both c and c++, certain community members immediatly jump on the question with comments like "there is no such language, pick one". To me, their further conduct comes across as un...

 
This site uses MathJax formatting of formulas. Consider adding a tag for a broader subject area to which the question belongs. Some of these tags might fit. More tips here. (from a bot)Normal Human 20 secs ago
0
Q: There exists c such that $\int_0^1 x^2 \! f(x) \, \mathrm{d}x = 1/3 f(c)$

user242520Let $f:[a,b]\rightarrow\mathbb{R}$ be a continuous function. Show that there exists $c\in[0,1]$ such that $\int_0^1 x^2 \! f(x) \, \mathrm{d}x = 1/3 f(c)$ I am stuck with this one. $f$ is said to be continuous on [a,b] so how can we conclude about [0,1]? Also can you please give me a clue abou...

0
Q: Finding area of rectangle under a parabola asymmetrical with respect to the Y-axis: What did I do wrong?

BramastaI am using these as references: How to find the dimensions of a rectangle if its area is to be a maximum? Does the symmetry of a parabola in finding the maximum area of a rectangle under said parabola matter? (related) Problem: Find the maximum area of a rectangle inside a parabola whose e...

0
Q: how does (n+1)! - 1 + (n+1)(n+1)! = (n+1)!(1+n+1) - 1

Matt.DoeI cannot figure this out help. This deals with Mathmatical induction problem. I've tried factoring but it doesn't work out.

0
Q: Is Z[−11−−−−√]Z[−11] factorial ring?

programmer0I cannot find a number that has two representations in this ring, please help

Short question. Question contains please. [Is Z[−11−−−−√]Z[−11] factorial ring?](math.stackexchange.com/q/1575480)
0
Q: weighted graph problem

A_lA graph with n nodes "shortest" is the sum of the edges weight of the shortest path tree to the root.minimal is the sum of the edges weight in MST. How Can I find the upper bound of shortest/minimal??

Short title. Title contains problem. weighted graph problem
0
Q: Lagrange Multipliers optimization

SademLet the manifold $S$ in $R^n$ be defined by $g(x)=0$. If $p$ is a point not on $S$, and $q$ is the point of $S$ which is closest to $p$, show that the line from $p$ to $q$ is perpendicular to $S$ at $q$, Hint: Minimize $f(x)=|x-p|^2$ First off I don't understand why that is the function to minim...

0
Q: Number of cycles and sign

MathnteaFor an even $n \in \mathbb{N}$, the sign of $\tau \in S_n$ is 1 if and only if the number of disjoint cycles in $\tau$ is even? And it is odd for odd $n$?

Short title. Short question. Number of cycles and sign
0
Q: Statistics question - quiz

danielThe diameter of the planet Earth is about 8000 miles. Imagine someone has tied a rope around the Earth ar the equator. You come along and as a practical joke put yardsticks all the way around the Earth and prop up the rope all the way around the equator. Estimate IN FEET to within 5% how much rop...

Welcome to Math.SE, daniel. Words such as question do not add information to titles. Please edit the title so that it better describes the specifics of your question. Do not hesitate to make it longer or include a formula if needed. More tips here. (from a bot)Normal Human 20 secs ago
0
Q: Max Modulus principal question

CaydeLet $f$ be an entire function such that $if(z)= \bar f(z)$ for all $\Bbb C$. Show that $f$ is the constant function. I didn't understand it but I took some notes which seem like gibberish to me now. Maybe someone could clarify: $\bullet$ I had drawn a line on a graph that has an arg of $\pi/4$...

Words such as question do not add information to titles. Please edit the title so that it better describes the specifics of your question. Do not hesitate to make it longer or include a formula if needed. More tips here. (from a bot)Normal Human 20 secs ago
0
Q: Dealing with questions on vectors and matrices with variables

FatratI'm running into a lot of questions on linear algebra lately where, instead of using constant numbers in matrices and vectors (these are questions which I have no problems in answering), they use a mix of constants and a variable a. I'm finding these difficult to answer. For example: 1) Determin...

This site uses MathJax formatting of formulas. More tips here. (autocomment)Normal Human 20 secs ago
0
Q: Dividing factorials

darrenI have (n+1)! / (n+2)! which simplifies to 1/(n+2) But I dont understand how this works Could someone explain the theory of factorials divide like this ?

Short title. Short question. Dividing factorials
0
Q: Checking if estimators are sufficient

user46697For an i.i.d. sample of random variables Xi distributed according to a normal distribution, known variance. I found a sufficient statistic—the sample mean. How do I check if other statistic like (X1+2X2)/3 or ∑Xi/(n+1) are also sufficient. Thank you.

Please don't use (self-learning) tag just because you were self-studying. This tag is only for questions about the process of self-studying. (from a bot)Normal Human 20 secs ago
0
Q: Problem with $arg(\gamma (t))$

WillyfI am see my notes about curves on complex spaces and I do not understand why it is so... I need help. I have this example: My notes said that I can defined $\theta: [a,b] \rightarrow \mathbb{R} /$ $\theta(t)=arg(\gamma(t))$ when $a\leq t \leq t_1$ $\theta(t)=arg(\gamma(t))+2\pi$ when $t_1< t

Title contains problem. Problem with $arg(\gamma (t))$
0
Q: Roots of real polynomials

alex alexeqI do not know how to deal with this problem: show that a real polynomial in two variables which does not change sign has only a finite number of roots

Short title. Short question. Roots of real polynomials
0
Q: How would you solve this paradox?

Ante PaladinThree friend walk into a bar, sit down, and when waiter comes to them, they ask him how much is the beer. The waiter tells them that the beer is $10$ dollars. They order three beers and each of them gives him $10$ dollars. A few moments later the boss of the bar arrives and asks waiter how much ...

0
Q: Prove that $x^y \equiv x^z \pmod p$ if and only if $y \equiv z \pmod p$

user4201961Is that true? $x^y \equiv x^z \pmod p$ if and only if $y \equiv z \pmod p$ If yes, then how to prove it?

Consider adding a tag for a broader subject area to which the question belongs. Some of these tags might fit. (autocomment)Normal Human 21 secs ago
 
6:03 PM
-3
Q: Should we handle flags more carefully..?

T JI asked this question here few days ago. It was about an Not an answer flag declined sighting: flags should not be used to indicate technical inaccuracies, or an altogether wrong answer. Later I came across this question and this answer. There was an issue with the data provided initially...

 
0
Q: How better (%) is A than B?

ntrxThis may sound stupid but, is this correct?: Is 100 a hundred percent "better" than 50? (Better as higher or something like that) If it is correct: How "better" is 50 than 30? and How much "worse" is 30 than 50? Don't kill me, please

Welcome to Math.SE, ntrx. Tag (soft-question) should not be the only tag a question has. Please add a tag for a subject area to which the question belongs. (autocomment)Normal Human 21 secs ago
0
Q: Calculate $\underset{n\rightarrow\infty}{lim}\cfrac{n^{2}}{\left(2+\frac{1}{n}\right)^{n}}$

Shahar ShalevHey need help to calculate : $\underset{n\rightarrow\infty}{lim}\cfrac{n^{2}}{\left(2+\frac{1}{n}\right)^{n}}$

0
Q: Plotting error function using matlab

ProbabilityGuyBasically, I have written codes for composite trapezoidal, and simpson numberical integration rules. I want to now to do graphs for the error (Number of rectangles vs error) of each of these methods. Is there a built-in function that do this in matlab? or do we have to write the code for it?

0
Q: Projective space, immersion, embeding

user289139How to check if transformation $F:\Bbb{P}^2\to \Bbb{P}^5$, given as follows $F([x,y,z])=([x^2,y^2,z^2,xy,yz,zx])$, is smooth, immersion or embedding?

0
Q: The width of a right triangle is twice its height. Find all angles in the triangle.

VmariaThe questions is: The width of a right triangle is twice its height. Find all angles in the triangle. How do I proceede? :)

0
Q: Probability of Winning the Lottery

Juggling_ProLets say there are 200 tickets in a lottery system. Six tickets are drawn one by one, without replacement. These six tickets win a prize. Lets say you bought 1 ticket, what is the probability that you win a prize? The obvious answer would just be 6/200 = 3%, but isn't it more than that since the...

0
Q: Distribution of ratio of functions of random variables

charnConsider $Y = \frac{1-\sin(2X)}{1-\sin(X)}$, where $X$ is a random variable. How to find the distribution of Y?

Welcome to Math.SE, Juggling_Pro. This site uses MathJax formatting of formulas. More tips here. (autocomment)Normal Human 21 secs ago
0
Q: Confusion on the proof for finite intersections of open sets being open

RJTKI have looked at the following source here (part (b)) for this proof. But I have a hard time seeing that it is completely correct. If we take $r = min(r_1, r_2)$ as in the reference how can we be sure that $B_r(x) \subset A_1 \cap A_2$ What if the portion of $A_1$ or $A_2$ that we needed for t...

Welcome to Math.SE, user298691. A title should not be all-MathJax; having some plain text helps with search and navigation. (autocomment)Normal Human 21 secs ago
0
Q: Statistics: Confidence intervals

BobFrom a sample of 1751 army hospitals, estimate the mean expenses for a full time equivalent employee for all US army hospitals using a 90% confidence interval given x = 6563 and s = 2484. Work: 1.645(2484/41.845) 1.645(59.362) 97.65 6563+- 97.65 6465.35---6660.65 ANswer I think I did that o...

0
Q: Existence proof : $\forall x\in \Bbb R \wedge x\geqq-3 \ \ \exists y \in \Bbb R : \frac{y(y-6)}{3}=x \\ $

user108410Prove that for every real number x with $x\geqq -3$ there exists a real number y such that $ \frac{y(y-6)}{3}=x $ $\forall x\in \Bbb R \wedge x\geqq-3 \ \ \exists y \in \Bbb R : \frac{y(y-6)}{3}=x \\ $ To prove this directly I must solve the equation for y and then plug the result into the le...

0
Q: Using integral test for convergence

darrenHow do I show that my series "the sum from n=1 to infinity of 1/n^3" is convergent using the integral test ?

0
Q: Why does a quaternion rotation matrix simplify to this?

ChrisI'm reading Ken Shoemake's explanation of quaternions in David Eberly's book Game Physics. In it, he defines the rotation matrix for a quaternion $q = x\mathbf{i} + y\mathbf{j} + z\mathbf{k} + w\mathbf{1}$ to be the product of two matrices: $\begin{align} \mathbf{Q\overleftarrow{Q}}^T &= \begin...

Questions tend to get more attention when they have a tag for a broad area of mathematics relevant to the question. Some of these tags might fit. (from a bot)Normal Human 21 secs ago
0
Q: Total confusion about differential one-forms and non-coordinate bases

Peter4075I asked this question recently (Basis of differential one-form confusion), thought I understood the answer, but now realise I don't. Lee (Introduction to Smooth Manifolds) says that at a point $p$ and with a vector field $X$ we define a covector field $df$, called the differential of $f$, b...

 
6:53 PM
0
Q: I'm not new to programming. I'm not bad at google. How do I avoid being considered stupid for asking "stupid" questions?

Seph Reed"Basically, the things I tried first, and the things I read first, were what everyone thought I was too stupid to find or read." The case: I want to know what swift programming language is all about, why I should learn a new one, what it might replace, what's broken in it, what it fixes. All t...

 
0
Q: Why isn't the fundamental theorem of line integrals applicable here?

Jake ChristensenThe question is as follows: Given vector field $$V = \left(\frac{1-y}{x^2 + (y-1)^2}, \frac{x}{x^2+(y-1)^2}\right)$$ Evaluate $$\int_{l_1}V \bullet dr\text{, }\int_{l_2}V \bullet dr$$ Where $l_1$ and $l_2$ are given as $$l_1: x^2+(y-1)^2=1\text{, } l_2: x^2+(y-4)^2=1$$ According to the ...

Questions tend to get more attention when they have a tag for a broad area of mathematics relevant to the question. Some of these tags might fit. (autocomment)Normal Human 20 secs ago
0
Q: How can we prove Q(sqrt(2), sqrt(3), ..... , sqrt(n) ) = Q(sqrt(2) + sqrt(3) + .... + sqrt(n) )

nicksohnI want to prove this statement. Q(sqrt(2), sqrt(3), ..... , sqrt(n) ) = Q(sqrt(2) + sqrt(3) + .... + sqrt(n) ) for any n >1 It looks very hard problem. How can i approach this one? Thanks!

This site uses MathJax formatting of formulas. More tips here. (from a bot)Normal Human 20 secs ago
0
Q: Cantor set is uncountable ( Independent of ternary expansion)

Anurag JainCan anybody please provide me proof of Cantor set is uncountable which is independent of ternary expansion. Thanks in advance.

Short question. Question contains please. Cantor set is uncountable ( Independent of ternary expansion)
0
Q: Modified Bessel differential equation

BowParkThe modified Bessel differential equation is always presented as $$r^2 \frac{\partial^2 f(r)}{\partial r^2} + r\frac{\partial f(r)}{\partial r} - (r^2 + n^2)f(r) = 0$$ with solutions $$f(r) = AI_n(r) + BK_n(r)$$ But if I had $$r^2 \frac{\partial^2 f(r)}{\partial r^2} + r\frac{\partial f(r)}{...

Tagged differential-equations but mentions "partial". Modified Bessel differential equation
0
Q: Proof Of Ergodic Theorem Problem

lkjhgfdsaI have been assigned to do the question I've attached. I have managed to do a,b, and c. Now I have 2 questions: (I'll use normal brackets for inner product brackets) Firstly, in part (a), I used that lim as n-->∞ of (An(x)-A(x)|y) = ∞ of: (An(x)-A(x)|y), and I don't know how to prove this contin...

Title contains problem. Proof Of Ergodic Theorem Problem
0
Q: EXP formula in excel using e

Saad DarCan someone explain what this formula is doing? =EXP(x)/(1+EXP(x))*100 If you are not familiar with EXP, EXP calculates the e^x. Thanks

Short title. Short question. EXP formula in excel using e
 
7:12 PM
0
Q: Evaluating a 2-form

FACEIT This is from Spivak Calculus on Manifolds, section 5.3 I have done part a, but I am stuck on part (b) and have been for a day now: let $p = (p_1,p_2,p_3)$ then $w(p)(v_p,w_p) = \dfrac{p_1 dy \wedge dz (v_p,w_p) + p_2 dz \wedge dx (v_p,w_p) + p_3dx \wedge dy (v_p,w_p)}{|p|^{3/2}}$ but I am uns...

Short title. Evaluating a 2-form
0
Q: Poisson Distribution #stat

PrecisionI posted early a bit a same question but i wanted to know if i'am doing alright another one. Let X be the number of customers buying a book in a bookstore e-shop. Assume X has a Poisson distribution with a mean of 1 books bought every 10 minutes. (a) What is the probability that no one will buy ...

0
Q: Numerical polynomial solver

user2350366I have fitted a polynomial surrogate model $y = f(x)$ to several sets of data and need to find the inverse of the function, i.e. $x = f^{-1}(y)$. The data is very well fit by polynomials - errors <1E-15. And all data is in the range [0,2]. However some of the polynomial surrogate models are quit...

0
Q: Proof of a congruence relation

Phenop is prime and a have a multipl. inverse in Z/pZ. prove: If a^(p-1)/2 ≡1 mod p then the congruence x^2≡a(mod p) has a solution x.

0
Q: Please help me solve real-analysis problem

ORLEONProblem: Assume we have the next recursive sequence: $x_n=\sqrt[3]{6+x_{n-1}}$, $x_1 = \sqrt[3]{6}$ Prove that exists such constant C (C $\neq$ 0) that: $\lim_{n\to+\infty}\frac{12^n(2-x_n)}{C}=1$ Can anyone help me with it?

Welcome to Math.SE, ORLEON. Words such as please, help are uninformative in titles. Please edit the title so that it better describes the specifics of your question. Do not hesitate to make it longer or include a formula if needed. More tips here. (autocomment)Normal Human 20 secs ago
0
Q: Use Cauchy Product to Prove sum of sine squared and cosine squared is 1

John Bradley WoodI know this solution has already been shown but each time I have seen examples of this proof I get lost at the last step. \begin{align} \cos^2 x + \sin^2 x &= \sum_{r = 0}^{\infty} \frac{(-1)^r}{(2r)!}\biggl(\sum_{k = 0}^r \binom{2r}{2k}(-1)^{2k}\biggr)x^{2r} + \sum_{r = 0}^{\infty} \frac{(-1)^r...

0
Q: Lie Bracket is not a tensor

AbellanSuppose that we have two vector fields $V,W \in TM$ with $M$ a differentiable manifold. We define the Lie Bracket as $[V,W](f)=V(W(f))-W(V(f))$ and it's easy to show that this bracket is indeed a vector field. I feel like I'm missing something but then why is not possible to define the Lie Brac...

0
Q: Some questions regarding NP-complexity

Daniel Paczuski BakI need to understand these proofs, but I haven't managed to find any material on these questions (especially the latter two). Any help proving these and explaining them would be awesome. Prove that P is in NP, Prove that polynomial-time reduction from NP-hard language L1 to L2 implies L2 is al...

Consider adding a tag for a broader subject area to which the question belongs. Some of these tags might fit. (autocomment)Normal Human 21 secs ago
 
7:45 PM
0
Q: I am getting a “You are currently banned from flagging…” message but no clear end for this ban is given

JakeGouldTitle says it all. I went to flag something as SPAM on the main Super User site recently and got a message (see screenshot below) that states, “You are currently banned from flagging…” Okay, fair enough. I understand declined flags can add up and result in a flagging ban. But in this case clear e...

 
0
Q: Laplace Transform with unit step function

David LundHere is the link to the question and the answer: https://gyazo.com/d98918d0e0eaabe88606c0314aff0aca Here is the link to what I have done: https://gyazo.com/20ddc3822c0c7bca888cd3334dd78281 What happens to 3PI? Where does it go?

0
Q: Inverse Limit of set

Shachar Orenif $limit_{n \to \infty} a_{1} + a_{2} + a_{3} + ... +a_{n}= L$ So, $limit_{n \to \infty} \frac{1}{a_{1}}+ \frac{1}{a_{2}} + \frac{1}{a_{3}} + ... + \frac{1}{a_{n}}= \frac{1}{L} $ ?

Short title. Inverse Limit of set
 
0
Q: Rescaling a picture when editing a post

HohmanfanIs there any simple way of changing the dimensions of a picture when adding it to a post? (Other than downloading and change it in an editor.) Sometimes the default size of the image I find is not good for the readability of what I am writing. //Unrelated, is meta the right place for this type o...

 
0
Q: Max price of a share

mkropkowskiCompany is planning to pay a dividend of 5\$ per share (dividend for previous year). Investor that wants to buy a shares of this company assumes that dividend will be stable (Thus will not change in future). What is the maximum price that investor can pay for one share, assuming that he will rece...

0
Q: Divisor of rational function

VincentI am finding the divisor of $f = (x_1/x_0) − 1 $on X, where X = $V ( x_1^2 + x_2 ^2 − x_0^ 2 ) ⊂ \mathbb P^ 2 $. Characteristic is not 2. I am totally new to divisor. So the plan in my mind is first find the prime divisor, and find ord(f) on every prime divisor. But I am having trouble in the fi...

Consider replacing (analysis) with a more specific tag for the relevant branch of analysis. (autocomment)Normal Human 27 secs ago
 
8:04 PM
0
A: Просьбы посоветовать книгу и т.п

QwertiyЕсли есть подходящий канонический ответ, то закрывать как дубликат. Если нет, то оставлять. Хоть какой-то контент наберётся, а потом можно будет и эталонный ответ собрать. А этот закрыть. Что касается ссылок, то для книг названия достаточно, чтобы найти, поэтому ссылка является просто дополнител...

0
Q: "Newest questions" button missing when returning from meta

HohmanfanWhen coming back to the main page from Meta, the usual questions show up, put the "newest" filter is missing. It re-appears if I click at the "questions" tab again. Is this a bug?

 
0
Q: Volume of the solid bounded below

GuestCalcUse a triple integral to find the volume of the solid bounded below by z = x 2+y 2 and bounded above by $$z = 8 − 2sqrt(x^2 + y^2) $$ How would you set up this integral?

0
Q: Determine the number of correct digits in the number $x$ given its relative error $E_r$

WarriorDetermine the number of correct digits in the number $x$ given its relative error $E_r$ (a): ~ $x=0.4785, E_r=0.2\times 10^{-2}$ (b) :~ $x=386.4, E_r=0.3$ (c): ~ $x=86.34, E_r=0.3$ For the problem (b), we have $E_r=0.3<0.5=\frac{1}{2}<\frac{1}{2} \frac{10}{4}=\frac{1}{2}\frac{1}{(3+1)\times ...

0
Q: Splitting Method

DoeConsider the iteration matrix for the general splitting method $M=I-N^{-1}A$ where $N$ is any invertible matrix. Show that if $\lambda =1$ is an eigenvalue of $M$. then $A$ cannot be invertible. I tried: $$Mx = \lambda x$$ $$(I-N^{-1}A)x = x$$ $$I-N^{-1}A = I$$ $$0 = N^{-1}A$$ But i'm not sur...

Short title. Splitting Method
0
Q: Logic formalization for Perfect Graph Matching problem

francescopA matching $M$ in a undirected graph $G(V,E)$ is a subset of the edges of $E$ such that no two edges in $M$ are incident to a common vertex. A perfect matching ${M}'$ is one in which every vertex is matched. Example Image So I want to express the set ${M}'$ with a mathematical-logic notation. ...

0
Q: raffle tomorrow , please help solve my problem?

trishaMy work is having a raffle tomorrow for a grand prize. 200 tickets have been sold, and there is only 1 Grand prize to win. I have bought six tickets. What are the odds I will win the prize?

Welcome to Math.SE, trisha. Words such as please, help are uninformative in titles. Please edit the title so that it better describes the specifics of your question. Do not hesitate to make it longer or include a formula if needed. More tips here. (autocomment)Normal Human 21 secs ago
0
Q: Question about Cauchy product of convergent series

RFZ 1) Since power series $\sum \limits_{n=0}^{\infty}a_nx^n$ converges at point $x=1$ then radius of convergence $R\geqslant 1$. Then $\sum \limits_{n=0}^{\infty}a_nx^n$ converges absolutely for $x<1$ since power series converges absolutely at any interior point of $(-R,R)$. 2) Hence $\sum \limits...

Words such as question do not add information to titles. Please edit the title so that it better describes the specifics of your question. Do not hesitate to make it longer or include a formula if needed. More tips here. (from a bot)Normal Human 21 secs ago
0
Q: Identity of a set of functions that rotates the Euclidean plane?

Niki MIf I have a set A consisting of all the functions f that rotate a plane by some number of degrees a E R, then what is the identity? Since the identity is supposed to be unique, should it be f that rotates the plane by 360 or f that rotates the plane by 0?

0
Q: Maximum reaminder $(a-1)^n+(a+1)^n\mod a^2$ for $3\le a\le 1000$

ADGHere's the problem: Let r be the remainder when (a−1)n + (a+1)n is divided by a2. For example, if a = 7 and n = 3, then r = 42: 63 + 83 = 728 ≡ 42 mod 49. And as n varies, so too will r, but for a = 7 it turns out that rmax = 42. For 3 ≤ a ≤ 1000, find ∑ rmax. Source What I...

Consider adding a tag for a broader subject area to which the question belongs. Some of these tags might fit. (from a bot)Normal Human 21 secs ago
0
Q: Find a family of open sets whose intersection is compact.

Narmina BaghirovaDoes such intersection exists? im thinking about $An=(3+1/n;4+1/n)$ since $\bigcap An = [3,4] $ so its closed and bounded then its compact. Can someone please say whether its correct or not?

Short question. Question contains please. Tagged proof-writing. Find a family of open sets whose intersection is compact.
0
Q: Exact Equations and Integrating Factor

M Arias I am really having difficulty with this problem. I understand part a, as I did d/dx[x^2y'+(x^2'-2x)y]=0 and then took the integral of both sides to get lny==x+2lnx+C. What I don't understand is how the integrating factor makes an equation exact and how to prove that.

This site uses MathJax formatting of formulas. More tips here. (from a bot)Normal Human 21 secs ago
0
Q: Test integral convergence

FogaI'm given the integral $$\int \limits_0 ^{+ \infty} \frac{e ^ {-\cos t} \cdot \sin (t ^ \beta)}{t^\alpha} dt \qquad a,b \in \mathbb{R}$$ and I need to test the absolute convergence. I split it in two parts, namely $$\int \limits_0 ^{1} \frac{e ^ {-\cos t} \cdot \sin (t ^ \beta)}{t^\alpha} dt + \...

0
Q: Suppose that R1 and R2 are reflexive relations?

ssn181234I need some help with this problem, Suppose that R1 and R2 are reflexive relations on a set A. Show that R1 ⊕ R2 is irreflexive?

 
9:02 PM
0
Q: What is invokescript tag?

zespriWhat is invokescript tag supposed to represent? When you click on it in the description area it reads "The invokescript tag has no usage guidance, can you help us create it?" Is this a useful tag? If yes, what is it for?

1
Q: My winter hat won't stay put

Daniel StormWhen viewing my profile with the iOS application my winter hat doesn't stay attached to my avatar. It's static in the top left corner as I continue to scroll through my profile. Example: Using the application on an iPhone 6S running iOS 9.2.

 
0
Q: given the polynomial what is the coefficient?

WatsonConsider the polynomial (−3x + 4y)^8. What is the coefficient of x^3y^5 in this polynomial? I need help clarifying this. When I create a Pascals Triangle I get the answer to be 56. Yet when my professor posted the answer to this problem, the answer was -8*7*27*1024.. How does that make any sense...

This site uses MathJax formatting of formulas. More tips here. (autocomment)Normal Human 21 secs ago
0
Q: Showing that $|\text{Hom}_\mathbb{Q}(K,K)|=6$

MiriamLet $g\in \mathbb{Q}[X]$ be irreducible with $\text{deg}\;g=3$. We assume that only one of the roots of $g$ is in $\mathbb{R}$. So, $\alpha \in \mathbb{R}$. Let $L\subset\mathbb{C}$ be the splitting field of $g$ over $\mathbb{Q}$. Show that for every homomorphism $\tau: \mathbb{Q}(\alpha)\right...

0
Q: How would you show that $ℝv=${$tv|t$in ℝ} is a subspace of $ℝ^n$?

CoolKidThis are my steps: $0$ is certainly in $ℝ^n$ If $at \in ℝ$, then $atv=t(av)=ℝ(av)=a(ℝv)$ If $t+s \in ℝ$, then $(t+s)v=tv+sv=ℝ(tv+sv)=(ℝt+ℝs)v$. Since I felt my steps are partly erroneous, could anyone fix it please?

0
Q: Is there a more intuitive way of coming up with matrices for problems like this? E.g. Find two matrices B and C with AB = AC, and B does not equal C.

heyheyheyyyyyLet A = $\begin{bmatrix}1 & 0\\1 & 0\end{bmatrix}$ Find two matrices B and C with AB = AC, and B does not equal C. I always have trouble with problems like this. Here, I know $\begin{bmatrix}1\\0\end{bmatrix}$ and $\begin{bmatrix}1\\1\end{bmatrix}$ works, but I've always done it with trial and ...

0
Q: The determinant is unique

NesaHow do I prove that the determinant is the only function that has the properties of the determinant (multilinearity,being alternating, $det(I)=1$). In my notes we started to prove it like: suppose there is another such function(say g)... But from then on I can't make sense of it.

0
Q: How do I calculate the error bound for a Maclaurin series?

AC255How many terms of the Maclaurin series of $f (x) = \ln(1 + x)$ are needed to compute ln(1.2) with an error of at most 0.0001?

 
9:35 PM
0
Q: strange review - "skip" as only option

guntbertHas anyone been presented with such a review?

 
0
Q: What is the limit of $v_n$?

1-approximationLet $x$ be nonnegative real number. The function $f_n$ is defined by: $$ \begin{align} f_n : &\mathbb{R}_{+}\mapsto\mathbb{R}\\ &x\phantom{R}\mapsto 3x^ne^{-x^2}-1. \end{align} $$ Let $v_n$ be a real number for all $n>1$ such that : $$ v_n >1,\;\text{and}\;f_n(v_n)=0. $$ Calculate the limit...

0
Q: Proof verification of linear independence of $\{1,x,x^2\}$

KuifjeI know that $\{1,x,x^2 \}$ is a linear independent family on $[-1,1]$, because $ax^2+bx+c$ has at most 2 roots, therefore if it equals $0$ for all $x$ of $[-1,1]$, it implies that $a=b=c=0$. However, I am wondering if the following proof is right or not: if $ax^2+bx+c=0$ on $[-1,1]$, it is true ...

 
9:54 PM
0
Q: Migrate [migrating] tag to [migration], [migrate], and [database-migration]

Joe KennedyI stumbled upon the migrating tag today; it has 62 questions, no tag wiki or usage guideline, and looking at the questions in that tag, no common theme other than a variation of the word "migrate". There are three tags that I think these questions can be moved into, migration, migrate, and databa...

0
Q: How to filter best (most voted) in last month/week/since last visit

jasirI would like to read the best questions in some time for my favorite tags, is there a way how to filter it?

 
0
Q: Find the volume of the region

GuestCalcThe region in R 3 bounded by the planes y = 1, y = −x, z = −x and, the coordinate planes, x = 0 and z = 0 $$ \int_{0}^{1} \int_{0}^{-y}\int_{0}^{-x} \;dz \;dx\;dy $$ I feel like I am a bit off with the bounds here? Can someone please confirm?

Short title. Question contains please. Find the volume of the region
 
10:09 PM
0
Q: server-client-server-client-server-... Let's avoid infinite recursions

user1803551server-client has a wiki and 63 questions, client-server has no wiki and 21 questions. I think that there is no real difference between them one should be merged into the other (probably the latter into the first).

 
0
Q: Formulation of matrix for FEM

user31765I am trying to use FEM to approximate the solution to the following BVP: $-\frac{d}{dx}[a(x)u'(x)]+b(x)u(x)=f(x)$, on [0,1] where $u(0)=0$ and $u(1)=1$. I am using the Galerkin method with hat functions: ${\varphi _j}$$\left( x \right)$ = $\left\{ {\begin{array}{*{20}{c}}{\frac{{x - {x_{j - 1}}...

 
10:31 PM
0
Q: Sequence with bounded l2 norm

MortonLet $f_i \colon [0,1]\rightarrow \mathbb{R}$ be a sequence of functions which converge to $f_\infty$ pointwise. How can I prove that $\lim_{i\rightarrow \infty} \int f_i d\lambda= \int f_\infty d\lambda$, when $||f_i||_2\leq 1$? My attemp is to use Egorov and then try to use the bound on the $l...

0
Q: Integrate 1/(x^3-25x)

Matthew BardonIntegrate 1/(x^3-25x) Between limits of 4,3. Think it involves the 'Difference in squares' substitution. I ended up with an answer of 0.3242 which is wrong as it is 0.023. Any help and pointers would be greatly appreciated. Matt

 
0
Q: Make [webusercontrols] and [web-user-controls] synonyms of [webusercontrol]

MPelletierwebusercontrol : 239 web-user-controls : 107 webusercontrols : 19

0
Q: when Reviewing : 'Suggested Edits' there is a noticable delay only for intentional Spam entries

Scott Stenslandits an observation ... see Title ... lately I see it repeatedly and so is a giveaway that you are about to be given an intentionally Spam entry when reviewing 'Suggested Edits' stackoverflow performance issue ?

 
0
Q: How small must h be for the trapezoidal rule to attain an error less than 0.001

JanetHornefor $\int x^2sin(x) \mathrm{d}x$ with x = 0.2 and 2.87 I am not exactly sure how to approach this problem.

0
Q: How to start learning diffenrential equations again?

caballerooopI finished my engineering career (Electronic Engineering), but i feel that my mathematical knowledge is deficient in important areas (like Fourier series, Laplace, differential equations.. etc). I'm willing to learn all over again mathematics, but i don't know where to start. Actually did try to...

Welcome to Math.SE, caballerooop. Questions tend to get more attention when they have a tag for a broad area of mathematics relevant to the question. Some of these tags might fit. (autocomment)Normal Human 20 secs ago
0
Q: Would like a hint for this graphical proof exercise on The Incredible Proof Machine

Michael WelchI'm going through all of the proofs in The Incredible Proof Machine and need a hint for one of the proofs. (The Incredible Proof Machine is an online graphical proof tool.) Given: $(\forall x.P(x)) \to A $ Prove: $\exists x.P(x) \to A $ It seems like a trivial proof and here's my hand-waving at...

0
Q: Possible pattern in primes

VictorThe function is quite simple and takes into account all integers > 2. It seems to spike the highest when the number is a prime, also numbers that are not primes are visibly under the prime line (red). Would this be worth looking into further? Thank you! Pattern

0
Q: Is this a bijection?

YakLet X be a set. All unions are disjoint unions. Suppose $f:X \rightarrow (\emptyset) \cup X \cup X^2$ is a bijection. Is there a injection $g: X \cup X^3 \rightarrow X^2 \cup X^4$? My first idea is to notice $g$ is equal to $h: X \times (\emptyset) \cup X^2 \rightarrow X^2 \times (\emptyset) \...

Short title. Is this a bijection?
 
10:57 PM
0
Q: Working with ajax, why do new technologies need to be invented?

ajaxGuruI've noticed with angular, and polymer new tags need to be invented, and was wondering why can't you just use the technologies already existing to run the HTML. I mean the tags do the same thing as tags that already exist.

 
0
Q: Let $f : U \subset \mathbb{R}^m \to \mathbb{R}$ be differentiable at $a \in U$.

B. RivasProve that there are $\epsilon >0$ and $M>0$ s.t. $|h|<\epsilon \implies |f(y)-f(x)|<M.|y-x|.$ A hint is enough, thank you.

0
Q: Help me figure out how this equation was rearranged please.

user2537537I'm trying hard to figure out how (x-a)^2 + (y-b)^2 = r^2 can be written as y = b + sqrt(r^2 - (x-a)^2) it says that you’ll want to have y as a function of x I don't now how to make math symbols like squaring on this website like x^y

Words such as help, please are uninformative in titles. Please edit the title so that it better describes the specifics of your question. Do not hesitate to make it longer or include a formula if needed. This site uses MathJax formatting of formulas. More tips here. (from a bot)Normal Human 21 secs ago
0
Q: Double Asymptotic Triangle

samsonite Is this a doubly asymptotic triangle in the poincare half plane or a singly asymptotic triangle?

Short title. Short question. Double Asymptotic Triangle
0
Q: Newton's identities from the n=k case

NesaHow do I derive the identities for the nk cases from the n=k case?(Proving Newton's identities)

 
11:15 PM
0
Q: Can we use Meta for hat-farming?

WhaaaaaatSo, with WinterBash upon us all, there's one very important question that everyone is asking: How do I get more hats?! Can we use the Meta site to test our and try to get more hats (provided we delete the questions after)?

 
0
Q: For a homework, how do i use resolution to prove G

Krishna KalyanF1 = ∀x∃y R(x,y) F2 = ∀x (∀yR(x,y) -> ∃z S(x,z)) F3 = ∀x∀y∀z (R(x,y) -> (S(x,z)-> S(y,z))) G = ∀x∃yS(x,y) Using resolution show that F1,F2,F3 |= G. So far I have done the following a)Prenex form of the formula b)Skolem form of the formula 1= Prenex ∃x∀y R(x,y) 1= Skolem ∀y R(f(y...

0
Q: Problem unerstanding Andersons prove of Tuttes Theorem

TheWaveLadIn Andersons prove of Tuttes Theorem using induction, they choose a maximal $X \in V(G)$ with $$q(G-X) = |X|$$ where $q(G-X)$ denotes the number of odd components in $G-X$. I don't understand the following statement from the prove: The subgraph $G-X$ has no odd components, because if $S$ is a...

0
Q: What is the derivative of (2x)^(4x)?

MeowI know the answer is [4^(2x+1)][x^(4x)][(1+ln(2x))] but I do not understand how it is attained.

 
0
Q: undelete of question does not update the score

overexchangeOn SO, delete/undelete does not update the user's score accordingly. I tested with query. http://stackoverflow.com/questions/34277353/why-style-properties-are-not-updated-in-dom-element-img

 
This site uses MathJax formatting of formulas. More tips here. (autocomment)Normal Human 21 secs ago
0
Q: Find volume of a rotation

user_99An arc of a curve $y=\frac{|x|}{1+x^2}$ is rotating around $O_{x^{-}}$ axis. Find the volume of a shape after rotation. Local maximum of a function $y$ are $M_1(1,\frac{1}{2}),M_2(-1,\frac{1}{2})$. I don't understand how to find the volume of a shape. I know the limits of integration (integrati...

0
Q: How do you show col $A=$ span{$c$} and row $A=$ span{$r$} based on the following condition.

CoolKidLet $A=cR$ where $c\ne 0$ is a column in $ℝ^m$ and $r \ne 0$ is a row in $ℝ^n$. Prove col $A=$ span{$c$} and row $A=$ span{$r$}. Could you give me an approach?

 
11:34 PM
0
Q: Does Stackoverflow use the tags at all to suggest/find similiar questions when typing question title

RafWhen asking a question, tags are the last fields that get populate before posting the question. I like the fact that when typing the question title, Stackoverflow suggests similar/related questions by searching against what is already being typed in the question title (or maybe question body too)...

1
Q: How come the Community diamond user is wearing a hat?

Bill WoodgerJust been reviewing to see if I could hit an audit, and I did. Community diamond is wearing a hat. How did that happen? Who opted it in to Hats, and then chose which hat it should wear?

 
0
Q: Application of stoke's theorem to boundaries

coffestap2 I am having troubles from the last line. It says if form $\omega$ is exact then $\omega = dr$ I understand how they are using stoke's theorem however I don't understand why $$\int_{\partial S^1} r = 0$$ - could someone explain this please?

0
Q: Maple solving inequalities within a range

nous3kI have no troubles to solve an inequality using this command: solve(df(x) > 0, x) However, I need to solve it within a specified range (-1..1). Is that possible?

Welcome to Math.SE, nous3k. Questions tend to get more attention when they have a tag for a broad area of mathematics relevant to the question. Some of these tags might fit. (from a bot)Normal Human 21 secs ago
 
00:00 - 16:0016:00 - 00:00

« first day (42 days earlier)      last day (532 days later) »